[Physics] Application of angular velocity to Euler angles

angular velocityrotational-kinematics

According to a post here Angular Velocity expressed via Euler Angles you can express angular velocity from euler angles. If I choose Y-Z-Y as a rotation sequence the expression becomes.

$\theta_r, \theta_p, \theta_y$ = roll, pitch, yaw

$$
\vec{\omega} = \dot{\theta_r} \hat{y} + R_z(\theta_p)( \left( \dot{\theta_p} \hat{z} + R_y(\theta_y) \left( \dot{\theta_y} \hat{y} \right) \right)
$$

which becomes

enter image description here

according to this

where

enter image description here

which does not make sense.

does it make sense and does it still work in this case?

Best Answer

Suppose you have a Y-Z-Y scheme with a corresponding sequence of rotation angles $\theta_y$, $\theta_p$ and $\theta_r$.

After the first rotation (yaw), the 3×3 orientation matrix $\mathrm{E}_y$ and angular velocity vector $\vec{\omega}_y$ is

$$\begin{aligned} \mathrm{E}_y & = \mathrm{rot}(\hat{j}, \theta_y) & \vec{\omega}_y & = \dot{\theta}_y \left(\hat{j}\right) \end{aligned} \;\tag{1}$$

The above should be self-evident. Now consider the second rotation and the orientation matrix $\mathrm{E}_p$ and angular velocity vector $\vec{\omega}_p$. Since the local axes are rotated by the first rotation we have

$$\begin{aligned} \mathrm{E}_p & = \mathrm{E}_y \mathrm{rot}(\hat{k}, \theta_p) & \vec{\omega}_p & = \dot{\theta}_y \left( \hat{j} \right) + \dot{\theta}_p \left( \mathrm{E}_y \hat{k} \right) \end{aligned} \;\tag{2}$$

Finally, with the third rotation we extend this pattern to find the final orientation matrix $\mathrm{E}$ and the final rotation velocity vector $\vec{\omega}$

$$\begin{aligned} \mathrm{E} & = \mathrm{E}_p \mathrm{rot}(\hat{j}, \theta_r) & \vec{\omega} & = \dot{\theta}_y \left( \hat{j} \right) + \dot{\theta}_p \left( \mathrm{E}_y \hat{k} \right) + \dot{\theta}_r \left( \mathrm{E}_p \hat{j} \right) \end{aligned} \;\tag{3}$$

The last part is re-written as

$$\begin{aligned} \mathrm{E} & =\mathrm{rot}(\hat{j}, \theta_y)\mathrm{rot}(\hat{k}, \theta_p) \mathrm{rot}(\hat{j}, \theta_r) & \vec{\omega} & = \dot{\theta}_y \hat{j} + \mathrm{rot}(\hat{j}, \theta_y) \left( \hat{k} \dot{\theta}_p + \mathrm{rot}(\hat{k}, \theta_p) \hat{j} \dot{\theta}_r \right) \end{aligned} \;\tag{4}$$

This expands out to the following jacobian formulation

$$ \vec{\omega} = \begin{bmatrix} 0 & \sin(\theta_y) & -\sin(\theta_p)\cos(\theta_y) \\ 1 & 0 & \cos(\theta_p) \\ 0 & \cos(\theta_y) & \sin(\theta_p) \sin(\theta_y) \end{bmatrix} \pmatrix{ \dot{\theta}_y \\ \dot{\theta}_p \\ \dot{\theta}_r } \;\tag{5}$$

Related Question